Những câu hỏi liên quan
nguyen thi ha
Xem chi tiết
nguyen thi ha
21 tháng 5 2018 lúc 15:42

giúp mình nha ,mìnhcaanf ngay câu trả lời , bạn nào trar lòi đúng mính k cho nha

  _ Mộc Tùy Tâm .-.
21 tháng 5 2018 lúc 15:49

1,x×1/2 + 3/2 × x = 4/5

X × (1/2 + 3/2 ) = 4/5

X × 2 = 4/5

4/5 : 2 = x

X = 2/5

2,

Số tròn trăm lớn nhất có 4 chữ số là :9900

Số lớn có 4 chữ số là : 9999

Tổng 2 số là :

9900 × 2 = 19 800

Số bé là : 

19 800 - 9 999 = 9 801 

3,

A)4/15 × 7/9 + 4/15 × 2/9 = 4/15 ×(7/9 +2/9)

                                               = 4/15 ×1 = 4/15

B)13 / 19 × 23 /11 - 13/19 × 8/11 - 13 / 19 × 4/11

=13/19 × (23/11 - 8/11 - 4/11)

= 13/19 × 1

= 13/19

vodanh
Xem chi tiết
chu tuấn anh
Xem chi tiết
chu tuấn anh
12 tháng 2 2020 lúc 8:50

mọi người ơiii ! giải giúp em với ạ :(((

Khách vãng lai đã xóa
Phạm Bảo Châu (team ASL)
26 tháng 9 2020 lúc 19:32

1.

a) \(-\frac{15}{17}>-\frac{19}{21}\)

b)\(-\frac{13}{19}>-\frac{19}{23}\)

c)\(-\frac{23}{49}>-\frac{25}{47}\)

d)\(\frac{317}{633}>\frac{371}{743}\)

e)\(-\frac{24}{35}< -\frac{19}{30}\)

f)\(\frac{12}{17}< \frac{13}{18}\)

g) \(-\frac{17}{26}< -\frac{16}{27}\)

h) \(\frac{84}{-83}< -\frac{337}{331}\)

i) \(-\frac{1941}{1931}< -\frac{2011}{2001}\)

j) \(-\frac{1930}{1945}>-\frac{1996}{2001}\)

k) \(\frac{37}{59}< \frac{47}{59}\)

I) \(-\frac{25}{124}>-\frac{27}{100}\)

m) \(-\frac{97}{201}>-\frac{194}{309}\)

n) \(-\frac{189}{398}< -\frac{187}{394}\)

o) \(-\frac{289}{403}>-\frac{298}{401}\)

Khách vãng lai đã xóa
Mắm Tôm
17 tháng 7 2021 lúc 12:32

Mắm Tôm Nè UID FF 1104310266 

Khách vãng lai đã xóa
Dương Đức Duy
Xem chi tiết
Dương Đức Duy
18 tháng 11 2021 lúc 15:35

Ai đúng được k!

Khách vãng lai đã xóa
Đào Quang	Hiếu
18 tháng 11 2021 lúc 15:36

e5r5565778890900967655343433334544565676766

Khách vãng lai đã xóa
Đinh Bá Phong
18 tháng 11 2021 lúc 15:37

18, 29, 47

Khách vãng lai đã xóa
Thanh Hà Đỗ
Xem chi tiết
Nguyễn Lê Phước Thịnh
4 tháng 2 2023 lúc 14:49

a: =>19/23>19/x>19/29

=>\(x\in\left\{24;25;26;27;28\right\}\)

b: =>88/132<88/x<88/128

=>132>x>128

=>\(x\in\left\{131;130;129\right\}\)

c: =>\(\left\{{}\begin{matrix}\dfrac{4}{x}-\dfrac{x}{8}< 0\\\dfrac{x}{8}-\dfrac{5}{x}< 0\end{matrix}\right.\Leftrightarrow\left\{{}\begin{matrix}\dfrac{32-x^2}{8x}< 0\\\dfrac{x^2-40}{8x}< 0\end{matrix}\right.\)

=>32<x^2<40

=>x=6

Peaceful Eden
Xem chi tiết
Nguyễn Ngọc Quý
7 tháng 8 2015 lúc 12:02

Bạn lấy mẫu chung

A=15/19 .17/29+15/29.19/23-15/19.13/23

A=15/19 (17/29 + 19/23 - 13/23)

A=15/19 . 1

A=15/19     

Haei
Xem chi tiết
Nguyễn Lê Phước Thịnh
10 tháng 5 2023 lúc 15:05

2: \(=\dfrac{-2}{75}+\dfrac{5}{39}=\dfrac{33}{325}\)

3: \(=\dfrac{6}{11}\left(\dfrac{4}{9}+\dfrac{5}{9}\right)=\dfrac{6}{11}\)

4: \(=\dfrac{7}{19}\left(\dfrac{5}{13}+\dfrac{8}{13}-1\right)=-2\cdot\dfrac{7}{19}=-\dfrac{14}{19}\)

5: \(=\dfrac{2}{7}\left(\dfrac{4}{23}-\dfrac{27}{23}+1\right)=0\)

6: \(=\dfrac{3}{8}\left(\dfrac{3}{7}+\dfrac{4}{7}\right)+\dfrac{11}{8}=\dfrac{3}{8}+\dfrac{11}{8}=\dfrac{14}{8}=\dfrac{7}{4}\)

Nguyễn Thùy Trang
Xem chi tiết
Nguyễn Thị Lan Anh
18 tháng 3 2017 lúc 20:28

A.5/7

B.-481/391

C.0

D.-59/9

E.1

tiểu thư họ nguyễn
18 tháng 3 2017 lúc 20:35

\(\dfrac{7}{-25}+\dfrac{18}{25}+\dfrac{4}{23}+\dfrac{5}{7}+\dfrac{19}{23}\)

=\(\left(\dfrac{-7}{25}+\dfrac{18}{25}\right)+\left(\dfrac{4}{23}+\dfrac{19}{23}\right)+\dfrac{5}{7}\)

= \(\dfrac{11}{25} +1+\dfrac{5}{7}\)

= \(1\dfrac{11}{25}+\dfrac{5}{7}\)

= \(2\dfrac{27}{175}\)

b) \(-2+\dfrac{15}{19}+\dfrac{-15}{17}+\dfrac{15}{23}+\dfrac{4}{19}\)

=\(-2+\left(\dfrac{ }{ }\right)\)

Tuệ Lâm
Xem chi tiết